Recent content by Kaldanis

  1. K

    Special Relativity, calculating velocity of Kl0 meson with reference frames

    Hm, but gamma requires v too? I also find this confusing.
  2. K

    Help with a triple integral in spherical coordinates

    Thank you! I was just talking with a friend and he also pointed out that I was using theta when I should be using phi. Now I have the correct answer and understand where I messed up. :smile:
  3. K

    Help with a triple integral in spherical coordinates

    If the first part is definitely correct, then I have to integrate \int^{\pi}_{0}\int^{2\pi}_{0} (2882/5)sin(θ)sin2(φ) dθdφ. Evaluating (2882/5)sin(θ)sin2 from 0 to 2π gives me zero, which means the final integral is also zero. Since the answer isn't zero I think my beginning equation must be...
  4. K

    Help with a triple integral in spherical coordinates

    I don't think this is right. It's a lot to write out in latex so I'll summarise it! \int^{\pi}_{0}\int^{2\pi}_{0}\int^{5}_{3} [(r sinφ cosθ)2+(r sinφ sinθ)2]r2sinθ drdθdφ = \int^{\pi}_{0}\int^{2\pi}_{0} (2882/5)sin(θ)sin2(φ) dθdφ This equals 0 so the final answer is 0? I always doubt...
  5. K

    Help with a triple integral in spherical coordinates

    Thank you both for explaining in detail, especially Tim. I'll go study it some more and come back with an answer for you to check. :)
  6. K

    Help with a triple integral in spherical coordinates

    Hi tiny-tim :smile: Should it be dρdθdφ? I've been trying to work through and learn from my book but it's very difficult to understand. I'm having trouble deciding which order to integrate things. Also I've noticed people using r instead of ρ but I don't think that matters to much, it's still...
  7. K

    Help with a triple integral in spherical coordinates

    Homework Statement Use spherical coordinates. Evaluate\int\int\int_{E}(x^{2}+y^{2}) dV where E lies between the spheres x2 + y2 + z2 = 9 and x2 + y2 + z2 = 25. The attempt at a solution I think my problem may be with my boundaries. From the given equations, I work them out to be...
  8. K

    Solving for L in Tunneling Probability Equation

    I'm still not having any luck with this problem. I know that I need to find the Potential Energy (U0) and total Energy. The work function is given as 5.1eV and from the picture it looks like this is the value of the PE, I think I could be wrong though. I still have no idea how to find potential...
  9. K

    Solving for L in Tunneling Probability Equation

    I'm in the process of studying for my final and I just can't solve this problem: The work function (energy needed to remove an electron) of gold is 5.1 eV. Two pieces of gold (at the same potential) are separated by a distance L. For what value of L will the transmission probability for an...
  10. K

    Help understanding/evaluating line integral over a curve

    Thank you everyone, the problem was that I was entering an approximate decimal value. It accepted \frac{8}{15}(1+391\sqrt{17}) as the correct answer!
  11. K

    Help understanding/evaluating line integral over a curve

    It's for online homework that only accepts the correct answer. I can only enter answers so many times before the question is locked, so I want to be sure of the answer before I enter it again
  12. K

    Help understanding/evaluating line integral over a curve

    Evaluate the line integral, where C is the given curve. \int_{c} xy\:ds, when C: x=t^{2}, \ y=2t\ , \ 0\leq t\leq4 To solve this I should use the formula \int^{b}_{a} f(x(t),y(t))\sqrt{(\frac{dx}{dt})^{2}+(\frac{dy}{dt})^{2}}dt This gives me \int^{4}_{0}...
  13. K

    Learning Calculus: What's the Right Order?

    I bought the Spivak book just before I started Calculus I so that I could 'prepare'. Huge mistake. I understood very little and to be quite honest, it made me fear what was about to come. I'm now taking Calc III and still won't go back to Spivak because of my earlier experiences with it. If...
  14. K

    Require help understanding small angle approximation

    Thank you, I had been using the equation incorrectly without realising. I have the correct answers now.
Back
Top